数学物理学报, 2023, 43(1): 14-26

Laplace算子特征值和的精细下界

何跃,1,3,*, 阮其华,2,3

1南京师范大学数学科学学院数学研究所 南京 210023

2莆田学院数学系 福建莆田 351100

3应用数学福建省高校重点实验室(莆田学院) 福建莆田 351100

Refined Lower Bound for Sums of Eigenvalues of the Laplace Operator

He Yue,1,3,*, Ruan Qihua,2,3

1Institute of Mathematics, School of Mathematics Sciences, Nanjing Normal University, Nanjing 210023

2Department of Mathematics, Putian University, Fujian Putian 351100

3Key Laboratory of Applied Mathematics (Putian University), Fujian Province University, Fujian Putian 351100

通讯作者: *何跃, E-mail: heyue@njnu.edu.cn; heyueyn@163.com

收稿日期: 2022-03-13   修回日期: 2022-08-15  

基金资助: 国家自然科学基金(11871278)
国家自然科学基金(11971253)
应用数学福建省高校重点实验室(莆田学院)开放课题(SX202101)

Received: 2022-03-13   Revised: 2022-08-15  

Fund supported: The NSFC(11871278)
The NSFC(11971253)
Key Laboratory of Applied Mathematics of Fujian Province University(Putian University)(SX202101)

作者简介 About authors

阮其华,E-mail:ruanqihua@163.com

摘要

该文研究了$\Bbb R ^n$中Laplace算子在有界域$\Omega$上的Dirichlet 特征值和的下界.众所周知:第$k$个Dirichlet特征值$\lambda_k(\Omega)$服从Weyl渐近公式,即 $ \lambda_k(\Omega)\sim\frac{4\pi^2}{[\omega_nV(\Omega)]^\frac{2}{n}}k^\frac{2}{n} \qquad\hbox{当}\,\,k\rightarrow\infty\,\,\hbox{时}, $ 其中$\omega_n$$V(\Omega)$分别为是$\Bbb R ^n$$n$维单位球的体积和$\Omega$的体积.根据上述公式,Pólya猜测 $ \lambda_k(\Omega)\geq\frac{4\pi^2}{[\omega_nV(\Omega)]^\frac{2}{n}}k^\frac{2}{n}, \quad\forall\,\,k\in{\Bbb N}. $ 这就是著名的Pólya猜想.对这一问题的研究由来已久,已有很多的工作.特别是,近几十年来最显著的成就之一是由Berezin[4], 以及李伟光和丘成桐[3] 分别独立取得的.他们部分解决了Pólya猜想,只是多了一个因子$n/(n+2)$.后来, Melas[7] 改进了Berezin-Li-Yau的估计,在不等式右边增加了一个正的$k$阶项. 该文采用与 Melas几乎相同的论证,进一步完善了 Melas 的估计.

关键词: (分数阶)Laplace算子; Dirichlet特征值; 高阶特征值; Weyl渐近公式; Pólya猜想; Berezin-Li-Yau不等式; 惯性矩

Abstract

In this paper, we study lower bounds for higher eigenvalues of the Dirichlet eigenvalue problem of the Laplacian on a bounded domain $\Omega$ in $\Bbb R ^n$. It is well known that the $k$-th Dirichlet eigenvalue $\lambda_k(\Omega)$ obeys the Weyl asymptotic formula, that is, $ \lambda_k(\Omega)\sim\frac{4\pi^2}{[\omega_nV(\Omega)]^\frac{2}{n}}k^\frac{2}{n} \qquad\hbox{as}\quad k\rightarrow\infty, $ where $\omega_n$ and $V(\Omega)$ are the volume of $n$-dimensional unit ball in $\Bbb R ^n$ and the volume of $\Omega$ respectively. In view of the above formula, Pólya conjectured that $ \lambda_k(\Omega)\geq\frac{4\pi^2}{[\omega_nV(\Omega)]^\frac{2}{n}}k^\frac{2}{n} \qquad\hbox{for}\quad k\in{\Bbb N}. $ This is the well-known conjecture of Pólya. Studies on this topic have a long history with much work. In particular, one of the more remarkable achievements in recent tens years has been achieved independently by Berezin[2] and Li and Yau[4], respectively. They solved partially the conjecture of Pólya with a slight difference by a factor $n/(n+2)$. Later, Melas[7] improved Berezin-Li-Yau's estimate by adding an additional positive term of the order of $k$ to the right side. Here, following almost the same argument as Melas, we further refine Melas's estimate.

Keywords: The (fractional) Laplace operator; The Dirichlet eigenvalue; Higher eigenvalues; The Weyl asymptotic formula; Conjecture of Pólya; The Berezin-Li-Yau inequality; The moment of inertia

PDF (407KB) 元数据 多维度评价 相关文章 导出 EndNote| Ris| Bibtex  收藏本文

本文引用格式

何跃, 阮其华. Laplace算子特征值和的精细下界[J]. 数学物理学报, 2023, 43(1): 14-26

He Yue, Ruan Qihua. Refined Lower Bound for Sums of Eigenvalues of the Laplace Operator[J]. Acta Mathematica Scientia, 2023, 43(1): 14-26

1 引言

$\Omega\subseteq\Bbb R ^n$是一个具有分片光滑边界$\partial\Omega$的有界域.考虑如下Dirichlet特征值问题

$\begin{equation}\label{dep-s-11-6-8-20-58} \left\{ \begin{array}{ll} -\Delta u=\lambda u,\qquad&\hbox{在}\,\,\Omega\,\,\hbox{中},\\ u=0,\qquad&\hbox{在}\,\,\partial\Omega\,\,\hbox{上}, \end{array} \right. \end{equation}$

其中$\lambda=\textrm{const}$.由于$-\Delta$的谱是实的和离散的,所以它的Dirichlet特征值能够以非递减的方式列出

$\begin{equation}\label{ineq-21-5-1-21} 0<\lambda_1(\Omega)<\lambda_2(\Omega)\leq\lambda_3(\Omega) \leq\cdots\leq\lambda_k(\Omega)\leq\cdots\nearrow\infty, \end{equation}$

其中每个$\lambda_j(\Omega)$是按照其有限重数重复列出的.

本文主要兴趣是集中于高阶特征值的估计方面.这里仅提到两个结果,一个是Weyl[1] 证明了渐近公式

$\begin{equation}\label{Weyl-asymptotic-formula} \lambda_k(\Omega)\sim\frac{4\pi^2}{[\omega_nV(\Omega)]^\frac{2}{n}}k^\frac{2}{n} \qquad\hbox{当}\,\,k\rightarrow\infty\,\,\hbox{时}, \end{equation}$

其中$\omega_n$$\Bbb R ^n$中单位球的体积.另一个是Pólya[2]证明了:如果$\Omega$是一个平铺平面区域,(1.3)式的左边不小于它的右边.(在某些文献中,这样的$\Omega$也称为一个平面覆盖区域,也就是说,它们全等的非重叠平移可以无间隙覆盖$\Bbb R ^2$).值得指出的是,Pólya的证明也同样适用于$\Bbb R ^n$中的平铺区域[3].根据Weyl的渐近公式及其结果,Pólya[2]还提出了以下猜想

Pólya猜想-. 设$\Omega$$\Bbb R ^n$中具有分片光滑边界$\partial\Omega$的有界区域. 那么问题(1.1)的第$k$个Dirichlet特征值$\lambda_k(\Omega)$满足

$\begin{equation}\label{Polya-conjecture} \lambda_k(\Omega)\geq\frac{4\pi^2}{[\omega_nV(\Omega)]^\frac{2}{n}}k^\frac{2}{n}. \end{equation}$

后来,上述猜想在许多文献中被广泛的研究(例如,[4-10]以及其中的参考文献).下面仅提到其中的几项重要成果.1980年,Lieb[5]证明了存在一个常数$c_n$使得

$\begin{equation}\label{Lie-1980-estimate} \lambda_k(\Omega)\geq c_n\frac{4\pi^2}{[\omega_nV(\Omega)]^\frac{2}{n}}k^\frac{2}{n}, \quad\forall\,\,k\in{\Bbb N}. \end{equation}$

然而,(1.5)式与(1.4)式相差一个因子$c_n(<1)$. 1983年, Li-Yau[3]在上述猜想中取得了很大进展.更准确地说,他们给出了如下估计

$\begin{equation}\label{Li-Yau-1983-esti-1} \sum_{j=1}^k\lambda_j(\Omega)\geq \frac{n}{n+2}\cdot\frac{4\pi^2}{[\omega_nV(\Omega)]^\frac{2}{n}}k^{1+\frac{2}{n}}, \quad\forall\,\,k\in{\Bbb N}. \end{equation}$

这在平均意义上是最佳的.值得一提的是,(1.6)式可以通过先前Berezin[4]的一个结果的Legendre变换得到

$ \sum_j(z-\lambda_j)_+^\sigma\leq L_{\sigma,n}^{cl}V(\Omega)z^{\sigma+\frac{n}{2}}, \quad\forall\,\,\sigma\geq1, $

其中 $ L_{\sigma,n}^{cl}:=\frac{\Gamma(\sigma+1)}{2^n\pi^\frac{n}{2}\Gamma(\sigma+1+\frac{n}{2})} $ 被称为Lieb-Thirring常数.这样,(1.6)式通常被称为Berezin-Li-Yau不等式. 由于$\lambda_1(\Omega)\leq\lambda_2(\Omega)\leq\cdots\leq\lambda_k(\Omega)$, 所以从(1.6)式可推出

$\begin{equation}\label{Li-Yau-1983-esti-2} \lambda_k(\Omega)\geq\frac{n}{n+2}\cdot\frac{4\pi^2}{[\omega_nV(\Omega)]^\frac{2}{n}}k^\frac{2}{n}, \quad\forall\,\,k\in{\Bbb N}. \end{equation}$

这显然部分解决了Pólya猜想,但多了一个因子$n/(n+2)$.

人们很自然地会问,是否可以在上式右边增加一个$k$的低阶的正项来改进(1.6)式. 沿着这个方向,Melas(2002)[7]迈出了实现这一目标的第一步,他得到了以下估计

$\begin{equation}\label{eqn-14-10-23a-21-38} \sum_{j=1}^k\lambda_j(\Omega)\geq \frac{n}{n+2}\cdot\frac{4\pi^2}{[\omega_nV(\Omega)]^\frac{2}{n}}k^{1+\frac{2}{n}} +\frac{1}{24(n+2)}\cdot\frac{V(\Omega)}{I(\Omega)}k, \quad\forall\,\,k\in{\Bbb N}, \end{equation}$

其中常数 $ I(\Omega):=\min\limits_{a\in\Bbb R ^n}\int_\Omega|x-a|^2{\rm d}x $ 就是所谓的$\Omega$的"惯性矩''.显然, 估计式(1.8)表明

$ \lambda_k(\Omega)\geq\frac{n}{n+2}\cdot\frac{4\pi^2}{[\omega_nV(\Omega)]^\frac{2}{n}}k^\frac{2}{n} +\frac{1}{24(n+2)}\cdot\frac{V(\Omega)}{I(\Omega)}, \quad\forall\,\,k\in{\Bbb N}. $

因此,这或多或少地改进了上面的Berezin-Li-Yau不等式.

关于这个专题的更多信息,可以在文献[6-7,9-12]以及其中的参考文献中找到.综上所述,谱几何中Laplace算子特征值和的研究一直是一个非常活跃的领域.

本文采用文献[7,9-10]的论证方法,但在细节上有一些显著的差异,证明了以下主要结果.

定理1.1$\Omega$$\Bbb R ^n\,(n\geq4)$中具有分片光滑边界$\partial\Omega$的有界区域.那么,对所有$k\in{\Bbb N}$, 问题(1.1)的特征值$\lambda_j(\Omega)\,(j\in{\Bbb N})$满足

$\begin{matrix}\label{main-ineq-(iii)} \sum_{j=1}^k\lambda_j(\Omega)&\geq& \frac{1}{n+2}\left\{\frac{4\pi^2n}{[\omega_nV(\Omega)]^\frac{2}{n}}k^{1+\frac{2}{n}} +\frac{5}{12}\cdot\frac{V(\Omega)}{I(\Omega)}k -\frac{5}{36}\cdot\frac{\omega_n^\frac{1}{n}V(\Omega)^{\frac{3}{2}+\frac{1}{n}}} {\pi I(\Omega)^\frac{3}{2}}k^{1-\frac{1}{n}}\right.\nonumber\\ &&\left.+\frac{29}{1536}\cdot\frac{\omega_n^\frac{2}{n}V(\Omega)^{2+\frac{2}{n}}} {\pi^2I(\Omega)^2}k^{1-\frac{2}{n}} -\frac{37}{38400}\cdot\frac{\omega_n^\frac{3}{n}V(\Omega)^{\frac{5}{2}+\frac{3}{n}}} {\pi^3I(\Omega)^\frac{5}{2}}k^{1-\frac{3}{n}}\right\}, \end{matrix}$

其中$\omega_n:=\pi^\frac{n}{2}/\Gamma(1+\frac{n}{2})$$\Bbb R ^n$$n$维单位球的体积, 而$\Gamma(x):=\int_0^\infty t^{x-1}e^{-t}{\rm d}t (x>0)$ 是 Gamma函数.

注1.1 使用与定理 1.1中几乎相同的论证,还得到以下结果

(i) 当维数$n\geq2$时,有

$\begin{equation}\label{main-ineq-(i)} \sum_{j=1}^k\lambda_j(\Omega)\geq \frac{1}{n+2}\left\{\frac{4\pi^2n}{[\omega_nV(\Omega)]^\frac{2}{n}}k^{1+\frac{2}{n}} +\frac{1}{8}\cdot\frac{V(\Omega)}{I(\Omega)}k -\frac{1}{72}\cdot\frac{\omega_n^\frac{1}{n}V(\Omega)^{\frac{3}{2}+\frac{1}{n}}}{\pi I(\Omega)^\frac{3}{2}}k^{1-\frac{1}{n}}\right\}. \end{equation}$

(ii) 当维数$n\geq3$时,有

$\begin{matrix}\label{main-ineq-(ii)} \sum_{j=1}^k\lambda_j(\Omega)&\geq& \frac{1}{n+2}\left\{\frac{4\pi^2n}{[\omega_nV(\Omega)]^\frac{2}{n}}k^{1+\frac{2}{n}} +\frac{1}{4}\cdot\frac{V(\Omega)}{I(\Omega)}k -\frac{1}{18}\cdot\frac{\omega_n^\frac{1}{n}V(\Omega)^{\frac{3}{2}+\frac{1}{n}}}{\pi I(\Omega)^\frac{3}{2}}k^{1-\frac{1}{n}}\right. \nonumber\\&& \left.+\frac{29}{7680}\cdot\frac{\omega_n^\frac{2}{n}V(\Omega)^{2+\frac{2}{n}}}{\pi^2I(\Omega)^2}k^{1-\frac{2}{n}}\right\}. \end{matrix}$

由于这些结果的证明几乎与定理1.1的相同,所以省略证明.

注1.2 从以下不等式(参见后面的(3.7)式)

$ n(n+2)B\geq n\tau^{n+2}+nI_n+2\sum_{i=0}^{n-1}(i+1)I_i\tau^{n-i}, $

可推导出

$\begin{equation}\label{main-ineq-1} n(n+2)B\geq \left\{ \begin{array}{lll} n\tau^{n+2}+2I_0\tau^n+4I_1\tau^{n-1},\quad&\forall\,\,n\geq2,\\ n\tau^{n+2}+2I_0\tau^n+4I_1\tau^{n-1}+6I_2\tau^{n-2},\quad&\forall\,\,n\geq3,\\ n\tau^{n+2}+2I_0\tau^n+4I_1\tau^{n-1}+6I_2\tau^{n-2}+8I_3\tau^{n-3},\quad&\forall\,\,n\geq4. \end{array} \right. \end{equation}$

本文将(1.12)中的不等式用于推导 Laplace算子特征值和的精细下界.这三种情况: (i) $n\geq2$, (ii) $n\geq3$和(iii) $n\geq4$ 的推理在总体上是相似的,它们之间的区别只出现在(1.12)式中.

注1.3(1.9)-(1.11)式确实改进了Melas的估计式(1.8).

注1.4 Yolcu等人[10]给出了以下结果

$\Omega$$\Bbb R ^n$中具有光滑边界的有界连通区域.对于$k\in{\Bbb N}$,且$1\leq\alpha\leq2$, $n=2$$0<\alpha\leq2$, $n\geq3$,分数阶Laplace算子$(-\Delta)^{\alpha/2}$ 的定义如下

$\begin{eqnarray*}\label{feac-laplace-eigenvalue} \left\{ \begin{array}{ll} (-\Delta)^{\alpha/2}\phi_j=\lambda_j^{(\alpha)}\phi_j, \qquad&\hbox{在}\,\,\Omega\,\,\hbox{中},\\ \phi_j=0,\qquad&\hbox{在}\,\,\Bbb R ^n\setminus\Omega\,\,\hbox{上} \end{array} \right. \end{eqnarray*}$

的特征值$\{\lambda_j^{(\alpha)}\}_{j=1}^\infty$满足

$\begin{eqnarray*} \sum_{j=1}^k\lambda_j^{(\alpha)}&\geq&(4\pi)^\frac{\alpha}{2}\frac{n}{\alpha+n} \left(\frac{\Gamma(1+\frac{n}{2})}{V(\Omega)}\right)^\frac{\alpha}{2}k^{1+\frac{\alpha}{n}}\\ &&+\frac{\alpha}{2(\alpha+n)}\frac{V(\Omega)^{\frac{1}{2}-\frac{\alpha-1}{n}}\Gamma(1+\frac{n}{2})^\frac{\alpha-1}{n}} {(4\pi)^{\frac{1}{2}-\frac{\alpha}{2}}I(\Omega)^\frac{1}{2}}k^{1+\frac{\alpha-1}{n}}\\ &&-\frac{5\alpha}{16(\alpha+n)}\frac{V(\Omega)^{1-\frac{\alpha-2}{n}}\Gamma(1+\frac{n}{2})^\frac{\alpha-2}{n}} {(4\pi)^{1-\frac{\alpha}{2}}I(\Omega)}k^{1+\frac{\alpha-2}{n}}\\ &&+\frac{\alpha}{16(\alpha+n)}\frac{V(\Omega)^{\frac{3}{2}-\frac{\alpha-3}{n}}\Gamma(1+\frac{n}{2})^\frac{\alpha-3}{n}} {(4\pi)^{\frac{3}{2}-\frac{\alpha}{2}}I(\Omega)^\frac{3}{2}}k^{1+\frac{\alpha-3}{n}}. \end{eqnarray*}$

然而,文献[10]给出的证明包含了一个错误的步骤.更具体地说,使用了下面不等式(参见文献[(53)],或文献[第8页])

$\begin{equation}\label{ineq-21-1-2-20} \int_{\tau}^{\tau+1}a(a-b)^{2}{\rm d}a\geq\min_{\tau\geq 0}\int_{\tau}^{\tau+1}a(a-b)^{2}{\rm d}a \geq\frac{1}{2}b^{2}-\frac{2}{3} b+\frac{1}{4}. \end{equation}$

但一般来说,这个不等式对于$b\geq1/2$的情形不成立.总之,由于证明存在瑕疵,所以他们的结果是不可靠的, 甚至可能是不正确的.这里仅仅指出,在$b\geq1/2$的情形,估计式(1.13)是错误的.例如,对于$b=1$$ \tau=\frac{1}{3}$,有

$ \int_{\tau}^{\tau+1}a(a-b)^{2}{\rm d}a=\int_\frac{1}{3}^\frac{4}{3}a(a-1)^{2}{\rm d}a =\frac{7}{9\times12}<\frac{1}{12}=\frac{1}{2}b^{2}-\frac{2}{3} b+\frac{1}{4}. $

本文剩余部分的安排如下:在第2节中,首先简要介绍一些记号,这与文献[7]中的记号是一致的,并为本文提供了一个框架; 在第3 节中,建立了一个关键引理,并给出了一个后面需要的重要推论; 在第4节中,借助上述推论,并采用文献[7,9-10]中类似的论证,证明了定理1.1.

2 记号和准备工作

这一节完全采用文献[7]中的记号,并叙述文献[4,7,9]中的一些后面用到的事实. 开集$\Omega$通过变换,可以假设

$ I(\Omega):=\min_{a\in\Bbb R ^n}\int_\Omega|x-a|^2{\rm d}x=\int_\Omega|x|^2\,{\rm d}x. $

$\{u_j\}_{j=1}^k$是相应于$\{\lambda_j(\Omega)\}_{j=1}^k$的一个标准正交特征函数族.$u_j$的Fourier变换可以表示为

$ \hat{u}_j(z):=(2\pi)^{-n/2}\int_\Omega u_j(x){\rm e}^{{\rm i}\langle x,z\rangle}{\rm d}x, $

其中$\langle\cdot,\cdot\rangle$表示$\Bbb R ^n$中的内积.定义如下的函数

$\begin{equation}\label{eq-21-1-1-1} F(z):=\sum_{j=1}^{k}\left|\hat{u}_j(z)\right|^{2}. \end{equation}$

引理2.1[4,7] 由(2.1)式定义的函数$F(z)$满足

$\begin{equation}\label{eq-21-1-2-16} \int_{\Bbb R ^{n}}F(z){\rm d}z=k, \end{equation}$
$\begin{equation}\label{eq-21-1-8-2} F(z)\leq(2\pi)^{-n}V(\Omega),\quad\forall\,\,z\in\Bbb R ^n, \end{equation}$
$\begin{equation}\label{eq-21-1-1-2} \int_{\Bbb R ^{n}}|z|^{2}F(z){\rm d}z=\sum_{j=1}^{k} \lambda_{j}(\Omega), \end{equation}$
$\begin{matrix}\label{eq-21-1-1-3} |\nabla F(z)|\leq2(2\pi)^{-n}\sqrt{V(\Omega)I(\Omega)}:=\rho,\quad\forall\,\,z\in\Bbb R ^n. \end{matrix}$

现在用

$\begin{equation}\label{eq-21-1-1-6} F^*(z)=\phi(|z|) \end{equation}$

表示$F$的径向递减重排列.(通过逼近$F$)可假设递减函数$\phi:[(2\pi)^{-n}V(\Omega)]$是绝对连续的.

引理2.2[7,9]$\phi$由(2.6)式定义.那么,

$\begin{equation}\label{ineq-21-1-1-5} 0\leq-\phi'(s)\leq\rho \end{equation}$

对几乎所有的$s$成立.

可以将(2.2)式重写如下

$\begin{equation}\label{eq(2.13)} k=\int_{\Bbb R ^{n}}F(z){\rm d}z=\int_{\Bbb R ^{n}}F^{*}(z){\rm d}z =n \omega_{n}\int_{0}^{\infty} s^{n-1}\phi(s){\rm d}s. \end{equation}$

由于$z\to|z|^2$是径向且递增的,由(2.4)式可以得到

$\begin{equation}\label{eq(2.14)} \sum_{j=1}^{k} \lambda_{j}(\Omega)= \int_{\Bbb R ^{n}}|z|^{2}F(z){\rm d}z\geq \int_{\Bbb R ^{n}}|z|^{2}F^{*}(z){\rm d}z =n \omega_{n} \int_{0}^{\infty} s^{n+1} \phi(s){\rm d}s. \end{equation}$

下面的初等结果,虽然简单但很重要,它建立了下面(3.1)式中出现的两个积分之间的联系.

引理2.3[7] 假设$h: [\infty)\to[0,1]$使得

$\begin{equation}\label{eq-21-21-1-2-2} \int_0^{\infty}h(s){\rm d}s=1\qquad\hbox{且}\qquad\int_0^{\infty}s^{n+2}h(s){\rm d}s<\infty. \end{equation}$

那么,存在$a\geq0$使得

$\begin{equation}\label{eq-21-1-2-9} \int_a^{a+1}s^n{\rm d}s=\int_0^{\infty}s^nh(s){\rm d}s\qquad\hbox{且}\qquad \int_a^{a+1}s^{n+2}{\rm d}s\leq\int_0^{\infty}s^{n+2}h(s){\rm d}s. \end{equation}$

改进特征值和的下界的关键之处是恰当使用下面的引理.它可以视为文献[引理4]的另一个版本.为了完整起见,这里给出了引理的证明.

引理2.4 对于整数$n\geq2$和正实数$s$$\tau$,有下列恒等式

$\begin{equation}\label{ineq-21-1-21-6} ns^{n+2}\equiv(n+2)\tau^{2}s^{n}-2\tau^{n+2} +\Big[ns^n+2\sum_{i=0}^{n-1}(i+1)\tau^{n-i}s^i\Big](s-\tau)^2. \end{equation}$

对于整数$n\geq2$,使用归纳法可得以下恒等式

$ nt^{n+2}\equiv(n+2)t^{n}-2 +\Big[nt^n+2\sum_{i=0}^{n-1}(i+1)t^i\Big](t-1)^2. $

$t=s/\tau$, 并将上述恒等式两边同时乘以$\tau^{n+2}$,可得(2.12)式.证毕.

3 技术性引理及推论

这一节将建立一个关键引理并给出其推论.后者在证明定理1.1中起着重要作用. 在这个过程中,使用与文献[7,9-10]相似但不完全相同的论证.读者可参考这些文献以获得更多细节.

引理3.1$n\geq4$, $A, B>0$$\psi: [0,\infty)\mapsto[0,\infty)$是一个递减的(且绝对连续的)函数,使得 $ \psi(0)=1,-1\leq\psi'(s)\leq0 $

$\begin{equation}\label{eq-21-5-13-15} \int_0^{\infty} s^{n-1}\psi(s){\rm d}s=A,\qquad \int_0^{\infty} s^{n+1}\psi(s){\rm d}s=B. \end{equation}$

那么,

$\begin{matrix}\label{eq-21-1-26-6} B\geq\frac{1}{n+2}\left[(nA)^{1+\frac{2}{n}}+\frac{5}{3}A -\frac{20}{9}n^{-\frac{1}{n}}A^{1-\frac{1}{n}} +\frac{29}{24}n^{-\frac{2}{n}}A^{1-\frac{2}{n}} -\frac{37}{150}n^{-\frac{3}{n}}A^{1-\frac{3}{n}}\right]. \end{matrix}$

可假设$B<\infty$, 否则就不需要证明了.显然$B<\infty$意味着$ \liminf_{t\to\infty}t^{n+2}\psi(t)=0$. 于是可得$ \liminf_{t\to\infty}t^{n+1}\psi(t)=0$.$h(s):=-\psi'(s),\forall s\geq0$.那么,$0\leq h(s)\leq1$

$\begin{equation}\label{eq-20-12-26-19} \int_0^{\infty}h(s){\rm d}s=\psi(0)=1. \end{equation}$

$ \liminf_{t\to\infty}t^{n+1}\psi(t)=0$和分部积分公式可得

$\begin{equation}\label{eq-20-12-26-21} \int_{0}^{\infty}s^{n}h(s){\rm d}s =\lim_{t\to\infty}\Big[-t^n\psi(t)+n\int_0^ts^{n-1}\psi(s){\rm d}s\Big] =n\int_{0}^{\infty}s^{n-1}\psi(s){\rm d}s=nA. \end{equation}$

由分部积分公式还可得到

$\begin{matrix}\label{ineq-20-12-29-6} \int_0^{\infty}s^{n+2}h(s){\rm d}s &=&\lim_{t\to\infty}\Big[-t^{n+2}\psi(t) +(n+2)\int_0^ts^{n+1}\psi(s){\rm d}s\Big]\nonumber\\ &=&(n+2)\int_0^{\infty}s^{n+1}\psi(s){\rm d}s =(n+2)B.\nonumber \end{matrix}$

根据引理2.3,存在$a\geq0$使得

$\begin{equation}\label{eq-20-12-26-8} \int_a^{a+1}s^n{\rm d}s=\int_0^{\infty}s^nh(s){\rm d}s=nA \end{equation}$

以及

$\begin{equation}\label{eq-20-12-22a-20} \int_a^{a+1}s^{n+2}{\rm d}s\leq\int_0^{\infty}s^{n+2}h(s){\rm d}s=(n+2)B. \end{equation}$

$\tau:=(nA)^{1/n}$. 显然,使用(3.5)式和Jensen不等式可得

$ \tau^n=nA=\int_a^{a+1}s^n{\rm d}s\geq\Big(\int_a^{a+1}s{\rm d}s\Big)^n =\big(a+\frac{1}{2}\big)^n\quad\Longrightarrow\quad \tau\geq a+\frac{1}{2}\quad\Longrightarrow\quad \tau-a\geq\frac{1}{2}. $

再次使用(3.5)式可得

$ \tau^n=nA=\int_a^{a+1}s^n{\rm d}s\leq(a+1)^n\quad\Longrightarrow\quad\tau\leq a+1 \quad\Longrightarrow\quad\tau-a\leq1. $

于是,有 $ \frac{1}{2}\leq\tau-a\leq1. $

现在引入记号

$ I_i:=\int_a^{a+1}s^i(s-\tau)^2{\rm d}s,\qquad i=0,1,2,3,\cdots. $

$[a,a+1]$上对(2.12)式积分,并使用(3.5)和(3.6)式可得

$\begin{matrix}\label{ineq-22-1-15-9-55} n(n+2)B&\geq&n\int_a^{a+1}s^{n+2}{\rm d}s\nonumber\\ &=&(n+2)\tau^{2}\int_a^{a+1}s^{n}{\rm d}s-2\tau^{n+2} +n\int_a^{a+1}s^{n}(s-\tau)^2{\rm d}s\nonumber\\ &&+2\sum_{i=0}^{n-1}(i+1)\tau^{n-i}\int_a^{a+1}s^i(s-\tau)^2s{\rm d}s\nonumber\\ &=&(n+2)\tau^{2}nA-2\tau^{n+2} +nI_n+2\sum_{i=0}^{n-1}(i+1)I_i\tau^{n-i}\nonumber\\ &=&n\tau^{n+2}+nI_n+2\sum_{i=0}^{n-1}(i+1)I_i\tau^{n-i}. \end{matrix}$

于是,容易看出

$\begin{equation}\label{ineq-21-5-13-15} n(n+2)B\geq n\tau^{n+2}+2I_0\tau^n+4I_1\tau^{n-1}+6I_2\tau^{n-2}+8I_3\tau^{n-3},\quad\forall\,\,n\geq4. \end{equation}$

接下来引入记号

$ J_j:=\int_a^{a+1}(s-a)^j(s-\tau)^2{\rm d}s,\qquad j=0,1,2,3,\cdots. $

利用Newton二项式定理,可进行如下计算

$\begin{matrix}\label{eq-21-6-1-18-1} I_i&=&\int_a^{a+1}\big[(s-a)+a\big]^i(s-\tau)^2{\rm d}s\nonumber\\ &=&\sum_{j=0}^i\bigg(\begin{array}{cc}i\\j\end{array}\bigg)a^{i-j}\int_a^{a+1}(s-a)^j(s-\tau)^2{\rm d}s =\sum_{j=0}^i\bigg(\begin{array}{cc}i\\j\end{array}\bigg)a^{i-j}J_j, \end{matrix}$

其中$\bigg(\begin{array}{cc}i\\j\end{array}\bigg)$表示通常的二项式系数,即 $ \bigg(\begin{array}{cc}i\\j\end{array}\bigg)=\frac{i!}{j!(i-j)!}. $

连续两次使用分部积分公式,可以继续进行如下计算

$\begin{eqnarray*}\label{} J_j&=&\frac{1}{j+1}\left\{(a+1-\tau)^2-\frac{2}{j+2}(a+1-\tau) +\frac{2}{j+2}\int_a^{a+1}(s-a)^{j+2}{\rm d}s\right\}\nonumber\\ &=&\frac{1}{j+1}\left\{(a+1-\tau)^2-\frac{2}{j+2}(a+1-\tau)+\frac{2}{(j+2)(j+3)}\right\}\nonumber\\ &=&\frac{1}{j+1}\left\{\left[(a+1-\tau)-\frac{1}{j+2}\right]^2+\frac{j+1}{(j+2)^2(j+3)}\right\}\nonumber\\ &\geq&\frac{1}{(j+2)^2(j+3)}. \end{eqnarray*}$

由上面的不等式,容易得到如下估计

$\begin{equation}\label{eq-20-12-26-22} I_0=J_0\geq\frac{1}{12},\quad J_1\geq\frac{1}{36}, \quad J_2\geq\frac{1}{80},\quad J_3\geq\frac{1}{150}. \end{equation}$

利用$a+1\geq\tau$,由(3.9)和(3.10)式可得如下估计

$\begin{matrix}\label{eq-21-1-12-13} I_1=aJ_0+J_1\geq\frac{a}{12}+\frac{1}{36} =\frac{a+1}{12}-\frac{1}{18} \geq\frac{\tau}{12}-\frac{1}{18}. \end{matrix}$

类似地,可以得到

$\begin{matrix}\label{ineq-21-1-26-10} I_2&=&a^2J_0+2aJ_1+J_2 \geq\frac{a^2}{12}+\frac{a}{18}+\frac{1}{80} =\frac{1}{12}\bigg[(a+1)^2-\frac{4}{3}(a+1)+\frac{29}{60}\bigg]. \end{matrix}$

定义函数$f:\Bbb R \mapsto\Bbb R $如下,

$ f(x):=x^2-\frac{4}{3}x+\frac{29}{60}=\big(x-\frac{2}{3}\big)^2+\frac{7}{180}, \quad\forall\,\,x\in\Bbb R. $

注意到$\frac{1}{2}\leq\tau\leq a+1$. 于是,可得

$ -\frac{1}{6}\leq\tau-\frac{2}{3}\leq a+\frac{1}{3}\qquad\Longrightarrow\qquad \big|\tau-\frac{2}{3}\big|\leq\max\big\{\frac{1}{6},a+\frac{1}{3}\big\}=a+\frac{1}{3}. $

从而可得

$\begin{equation}\label{ineq-21-2-2-2} f(a+1)=\big(a+\frac{1}{3}\big)^2+\frac{7}{180} \geq\big(\tau-\frac{2}{3}\big)^2+\frac{7}{180}=f(\tau). \end{equation}$

由(3.12)和(3.13)式可得如下估计

$\begin{matrix}\label{eq-21-1-26-12} I_2\geq\frac{1}{12}f(a+1)\geq\frac{1}{12}f(\tau) =\frac{1}{12}\big(\tau^2-\frac{4}{3}\tau+\frac{29}{60}\big). \end{matrix}$

类似地,通过直接计算可得

$\begin{eqnarray*}\label{eq-21-1-26-13} I_3&=&a^3J_0+3a^2J_1+3aJ_2+J_3 \geq\frac{a^3}{12}+\frac{a^2}{12}+\frac{3a}{80}+\frac{1}{150}\\ &=&\frac{1}{12}(a+1)^3-\frac{1}{6}(a+1)^2+\frac{29}{240}(a+1)-\frac{37}{1200}. \end{eqnarray*}$

定义如下的函数

$ g(x):=\frac{1}{12}x^3-\frac{1}{6}x^2+\frac{29}{240}x-\frac{37}{1200}, \quad\forall\,\,x\in[0,\infty). $

显然,容易验证

$ g'(x)=\frac{1}{4}x^2-\frac{1}{3}x+\frac{29}{240} =\frac{1}{4}(x-\frac{2}{3})^2+\frac{7}{720}>0, $

这意味着$g(x)$$[0,\infty)$上是递增的.于是,利用$a+1\geq\tau$可得

$\begin{equation}\label{ineq-21-1-28-1} I_3\geq g(a+1)\geq g(\tau) =\frac{1}{12}\tau^3-\frac{1}{6}\tau^2+\frac{29}{240}\tau-\frac{37}{1200}, \end{equation}$

这样,将(3.10),(3.11),(3.14)式和(3.15)式代入(3.8)式可得

$\begin{matrix}\label{eq-21-1-21-8} n(n+2)B&\geq&n\tau^{n+2}+\frac{1}{6}\tau^n +\left(\frac{1}{3}\tau-\frac{2}{9}\right)\tau^{n-1} +\left(\frac{1}{2}\tau^2-\frac{2}{3}\tau+\frac{29}{120}\right)\tau^{n-2}\nonumber\\ &&+\left(\frac{2}{3}\tau^3-\frac{4}{3}\tau^2 +\frac{29}{30}\tau-\frac{37}{150}\right)\tau^{n-3}\nonumber\\ &=&n\tau^{n+2}+\frac{5}{3}\tau^n-\frac{20}{9}\tau^{n-1} +\frac{29}{24}\tau^{n-2}-\frac{37}{150}\tau^{n-3}. \end{matrix}$

再将$\tau=(nA)^{1/n}$代入上面的不等式,可得到如下估计

$\begin{eqnarray*}\label{eq-21-1-21-18} B\geq\frac{1}{n+2}\left[(nA)^{1+\frac{2}{n}}+\frac{5}{3}A -\frac{20}{9}n^{-\frac{1}{n}}A^{1-\frac{1}{n}} +\frac{29}{24}n^{-\frac{2}{n}}A^{1-\frac{2}{n}} -\frac{37}{150}n^{-\frac{3}{n}}A^{1-\frac{3}{n}}\right]. \end{eqnarray*}$

证毕.

作为引理3.1的推论, 给出下边的结果.

推论3.1$n\geq4$, $\rho, A, B>0$$\psi: [0,\infty)\mapsto[0,\infty)$是一个递减的(且绝对连续的)函数,使得 $ -\rho\leq\psi'(s)\leq0, $

$\begin{eqnarray*}\label{eq-21-3-27-10} \int_0^{\infty} s^{n-1}\psi(s){\rm d}s=A,\qquad \int_0^{\infty} s^{n+1}\psi(s){\rm d}s=B. \end{eqnarray*}$

那么,

$\begin{matrix}\label{ineq-21-1-12-6} B&\geq&\frac{1}{n+2}\Big[(nA)^{1+\frac{2}{n}}\psi(0)^{-\frac{2}{n}} +\frac{5}{3}A\rho^{-2}\psi(0)^2 -\frac{20}{9}n^{-\frac{1}{n}}A^{1-\frac{1}{n}}\rho^{-3}\psi(0)^{3+\frac{1}{n}}\nonumber\\ &&+\frac{29}{24}n^{-\frac{2}{n}}A^{1-\frac{2}{n}}\rho^{-4}\psi(0)^{4+\frac{2}{n}} -\frac{37}{150}n^{-\frac{3}{n}}A^{1-\frac{3}{n}}\rho^{-5}\psi(0)^{5+\frac{3}{n}}\Big]. \end{matrix}$

$\widetilde{\psi}(t)=\alpha\psi(\beta t)$,其中$\alpha,\beta>0$是待定常数.显然,令$\alpha=\psi(0)^{-1}$,可得$\widetilde{\psi}(0)=1$. 此外, $ \widetilde{\psi}'(t)=\alpha\beta\psi'(\beta t). $ 于是,从$-\rho\leq\psi'(s)\leq0$可推导出 $ -\rho\alpha\beta\leq\alpha\beta\psi'(\beta t)\leq0. $ 再令$\beta=\rho^{-1}\psi(0)$可得$\rho\alpha\beta=1$.于是,可得$-1\leq\widetilde{\psi}'(t)\leq0$.$s=\beta t$$t=s/\beta$. 由此可得

$ A=\int_0^{\infty} s^{n-1}\psi(s){\rm d}s =\int_0^{\infty}(\beta t)^{n-1}\psi(\beta t)\beta{\rm d}t =\frac{\beta^n}{\alpha}\int_0^{\infty}t^{n-1}\widetilde{\psi}(t){\rm d}t, $

于是,有

$ \int_0^{\infty}t^{n-1}\widetilde{\psi}(t){\rm d}t=A\alpha\beta^{-n}:=\widetilde{A}. $

将(3.2)式应用到$\widetilde{\psi}$可得

$\begin{eqnarray*} \widetilde{B}&:=&\int_0^{\infty}t^{n+1}\widetilde{\psi}(t){\rm d}t\\ &\geq&\frac{1}{n+2}\Big[(n\widetilde{A})^{1+\frac{2}{n}}+\frac{5}{3}\widetilde{A} -\frac{20}{9}n^{-\frac{1}{n}}\widetilde{A}^{1-\frac{1}{n}} +\frac{29}{24}n^{-\frac{2}{n}}\widetilde{A}^{1-\frac{2}{n}} -\frac{37}{150}n^{-\frac{3}{n}}\widetilde{A}^{1-\frac{3}{n}}\Big]. \end{eqnarray*}$

于是,将$\widetilde{\psi}(t)=\alpha\psi(\beta t)$$\widetilde{A}:=A\alpha\beta^{-n}$代入上面不等式可得

$\begin{matrix}\label{ineq-21-1-2-21} \int_0^{\infty}t^{n+1}\alpha\psi(\beta t){\rm d}t&\geq& \frac{1}{n+2}\Big[(nA\alpha\beta^{-n})^{1+\frac{2}{n}}+\frac{5}{3}A\alpha\beta^{-n} -\frac{20}{9}n^{-\frac{1}{n}}(A\alpha\beta^{-n})^{1-\frac{1}{n}}\nonumber \\ &&+\frac{29}{24}n^{-\frac{2}{n}}(A\alpha\beta^{-n})^{1-\frac{2}{n}} -\frac{37}{150}n^{-\frac{3}{n}}(A\alpha\beta^{-n})^{1-\frac{3}{n}}\Big]. \end{matrix}$

通过变量变换:$s=\beta t$,(3.18)式的左边可改写为

$\begin{matrix}\label{ineq-21-1-2-22} \int_0^{\infty}t^{n+1}\alpha\psi(\beta t){\rm d}t =\alpha\beta^{-(n+2)}\int_0^{\infty}s^{n+1}\psi(s){\rm d}s. \end{matrix}$

注意到$\alpha=\psi(0)^{-1}$$\beta=\rho^{-1}\psi(0)$,并利用(3.18)和(3.19)式可得

$\begin{eqnarray*} \int_0^{\infty}s^{n+1}\psi(s){\rm d}s&\geq& \frac{1}{n+2}\Big[(nA)^{1+\frac{2}{n}}\alpha^\frac{2}{n}+\frac{5}{3}A\beta^2 -\frac{20}{9}n^{-\frac{1}{n}}A^{1-\frac{1}{n}}\alpha^{-\frac{1}{n}}\beta^3 \\ &&+\frac{29}{24}n^{-\frac{2}{n}}A^{1-\frac{2}{n}}\alpha^{-\frac{2}{n}}\beta^4 -\frac{37}{150}n^{-\frac{3}{n}}A^{1-\frac{3}{n}}\alpha^{-\frac{3}{n}}\beta^5\Big] \\ &=&\frac{1}{n+2}\Big[(nA)^{1+\frac{2}{n}}\psi(0)^{-\frac{2}{n}} +\frac{5}{3}A\rho^{-2}\psi(0)^2 -\frac{20}{9}n^{-\frac{1}{n}}A^{1-\frac{1}{n}}\rho^{-3}\psi(0)^{3+\frac{1}{n}} \\ &&+\frac{29}{24}n^{-\frac{2}{n}}A^{1-\frac{2}{n}}\rho^{-4}\psi(0)^{4+\frac{2}{n}} -\frac{37}{150}n^{-\frac{3}{n}}A^{1-\frac{3}{n}}\rho^{-5}\psi(0)^{5+\frac{3}{n}}\Big]. \end{eqnarray*}$
这证明了推论3.1.
注3.1 不等式(3.17)改进了Melas的结果(参见文献[引理1]).

4 定理1.1的证明

以推论3.1为基础,下面准备证明定理1.1.这个论证过程总体上与文献[7,9,10]是一样的,但又不完全相同.

函数$\phi$由(2.6)式所定义.将(3.17)式应用于$\phi$,且取

$ B=\int_0^{\infty} s^{n+1}\phi(s){\rm d}s,\qquad A=\frac{k}{n\omega_n}, \qquad\rho=2(2\pi)^{-n}\sqrt{V(\Omega)I(\Omega)}, $

并注意到

$ \frac{1}{n\omega_n}\sum_{j=1}^k\lambda_j(\Omega)\geq B, $

于是,可得

$\begin{matrix}\label{ineq-21-1-29-3)} \sum_{j=1}^{k}\lambda_j(\Omega)&\geq& \frac{1}{n+2}\Big[n\omega_{n}^{-\frac{2}{n}}k^{1+\frac{2}{n}}\phi(0)^{-\frac{2}{n}} +\frac{5}{3}k\rho^{-2}\phi(0)^{2} -\frac{20}{9}k^{1-\frac{1}{n}}\omega_n^\frac{1}{n}\rho^{-3}\phi(0)^{3+\frac{1}{n}}\nonumber\\ &&+\frac{29}{24}k^{1-\frac{2}{n}}\omega_n^\frac{2}{n}\rho^{-4}\phi(0)^{4+\frac{2}{n}} -\frac{37}{150}k^{1-\frac{3}{n}}\omega_n^\frac{3}{n}\rho^{-5}\phi(0)^{5+\frac{3}{n}}\Big]. \end{matrix}$

现在取$R:=\{V(\Omega)/\omega_n\}^{1/n}$使得$V\big(B_R(0)\big)=\omega_nR^n=V(\Omega)$.$B_R(0)$就是$\Omega$的对称重排. 这样,由$I(\Omega)$的定义可得

$ I(\Omega)\geq\int_{B_R(0)}|x|^{2}{\rm d}x=n\omega_n\int_0^Rs^{n+1}{\rm d}s =\frac{n\omega_{n}R^{n+2}}{n+2} =\frac{n}{n+2}\omega_{n}^{-\frac{2}{n}}V(\Omega)^{\frac{n+2}{n}}. $

于是,可直接验证

$\begin{matrix}\label{est-rho} \rho &\geq& 2(2\pi)^{-n}\sqrt{\frac{n}{n+2}\omega_{n}^{-\frac{2}{n}}V(\Omega)^{\frac{n+2}{n}+1}} =2\sqrt{\frac{n}{n+2}}\cdot(2\pi)^{-n}\omega_{n}^{-\frac{1}{n}} V(\Omega)^{\frac{n+1}{n}} \\ &\geq &(2\pi)^{-n}\omega_{n}^{-\frac{1}{n}} V(\Omega)^{\frac{n+1}{n}}. \end{matrix}$

注意到$0<\phi(0)\leq(2\pi)^{-n}V(\Omega)$.构造一个辅助函数如下

$ \theta(t):=\xi(t)+\eta(t)+\zeta(t),\quad\forall\,\,t>0, $

其中

$ \xi(t):=n\omega_{n}^{-\frac{2}{n}}k^{1+\frac{2}{n}}t^{-\frac{2}{n}} +\frac{5}{3}k\rho^{-2}t^{2}, $
$ \eta(t):=-\frac{20}{9}k^{1-\frac{1}{n}}\omega_n^\frac{1}{n}\rho^{-3}t^{3+\frac{1}{n}} +\frac{29}{24}k^{1-\frac{2}{n}}\omega_n^\frac{2}{n}\rho^{-4}t^{4+\frac{2}{n}} $

以及

$ \zeta(t):=-\frac{37}{150}k^{1-\frac{3}{n}}\omega_n^\frac{3}{n}\rho^{-5}t^{5+\frac{3}{n}}. $

然后,将证明$\xi(t)$,$\eta(t)$$\zeta(t)$$\big(0,(2\pi)^{-n}V(\Omega)\big]$上都是递减的. 这样,$\theta(t)$在这个区间上也是递减的.因此,在(4.1)式中可用$(2\pi)^{-n}V (\Omega)$代替$\phi(0)$,并将$\rho=2(2\pi)^{-n}\sqrt{V(\Omega)I(\Omega)}$代入这个不等式,从而得到所期望的估计.

为了证明这个定理,现在按如下步骤进行.容易看出$\zeta(t)$$(0,\infty)$上是递减的.由直接计算可得

$ \xi'(t)=-2\omega_{n}^{-\frac{2}{n}}k^{1+\frac{2}{n}}t^{-\frac{2}{n}-1} +\frac{10}{3}k\rho^{-2}t,\quad\forall\,\,t>0. $
$ \xi''(t)=2(\frac{2}{n}+1)\omega_{n}^{-\frac{2}{n}}k^{1+\frac{2}{n}}t^{-\frac{1}{n}-2} +\frac{10}{3}k\rho^{-2}>0, \quad\forall\,\,t>0, $

这表明$\xi(t)$$(0,\infty]$上是凸函数. 令$\xi'(t)=0$可得

$ t_0:=\Big[\big(\frac{3}{5}\big)^\frac{n}{2}\omega_n^{-1}k\rho^n\Big]^\frac{1}{n+1}. $

显然,$t_0$是使$\xi'(t)=0$的唯一点,且当$t\rightarrow0^+$时,$\xi'(t)\rightarrow-\infty$. 因此,$\xi'(t)\leq0,\forall\,t\in(0,t_0]$.另一方面,利用(4.2)式可得

$\begin{eqnarray*}\label{ineq-21-5-1516} t_0=\Big[\big(\frac{3}{5}\big)^\frac{n}{2}\omega_n^{-1}k\rho^n\Big]^\frac{1}{n+1} \geq\Big[\big(\frac{3}{5}\big)^\frac{n}{2}\omega_n^{-2}k(2\pi)^{-n^2}V(\Omega)^{n+1}\Big]^\frac{1}{n+1} \geq(2\pi)^{-n}V(\Omega). \end{eqnarray*}$

上述最后一个不等式等价于

$ k\geq\big(\frac{12}{5}\big)^{-\frac{n}{2}}\cdot\frac{\omega_n^2}{\pi^n}, $

这显然是对的.事实上,由于$\Gamma(s)$$[2,+\infty)$上是递增的,于是

$ \Gamma(1+\frac{n}{2})\geq\Gamma(2)=1. $

从而就有

$ k\geq1\geq\big(\frac{12}{5}\big)^{-\frac{n}{2}}\cdot\left[\Gamma(1+\frac{n}{2})\right]^{-2} =\big(\frac{12}{5}\big)^{-\frac{n}{2}}\cdot\frac{\omega_n^2}{\pi^n}. $

因此,由上面论证可知:在$(0,(2\pi)^{-n}V(\Omega)]$$\xi'(t)\leq0$,这意味着$\xi(t)$在这个区间上是递减的.

接下来,继续进行如下计算

$\begin{eqnarray*} \eta'(t) & = & -\frac{20}{9}\big(3+\frac{1}{n}\big)k^{1-\frac{1}{n}}\omega_n^\frac{1}{n}\rho^{-3}t^{2+\frac{1}{n}} +\frac{29}{24}\big(4+\frac{2}{n}\big)k^{1-\frac{2}{n}}\omega_n^\frac{2}{n}\rho^{-4}t^{3+\frac{2}{n}} \\ & = & \frac{29}{24}\big(4+\frac{2}{n}\big)k^{1-\frac{2}{n}}\omega_n^\frac{2}{n}\rho^{-4}\cdot t^{2+\frac{1}{n}} \big(t^{1+\frac{1}{n}}-t_*^{1+\frac{1}{n}}\big), \end{eqnarray*}$

其中

$ t_*:=\left[\Big(\frac{160}{87}\cdot\frac{3+\frac{1}{n}}{4+\frac{2}{n}}\Big)^nk\omega_n^{-1}\rho^n\right]^{\frac{1}{n+1}}. $

显然, $t_*\geq(k\omega_n^{-1}\rho^n)^{\frac{1}{n+1}}$,且 $ \eta'(t)\leq0,\ \forall\,\,x\in[t_*], $ 这意味着$\eta(t)$$[t_*]$上是递减的.另一方面,再次利用(4.2)式可得

$ t_*\geq(k\omega_n^{-1}\rho^n)^{\frac{1}{n+1}} \geq\big[\omega_n^{-2}k(2\pi)^{-n^2}V(\Omega)^{n+1}\big]^\frac{1}{n+1} \geq(2\pi)^{-n}V(\Omega). $

上述最后一个不等式等价于 $ k\geq(2\pi)^{-n}\omega_n^2, $ 这显然是对的.事实上,

$ k\geq1\geq2^{-n}\left[\Gamma(1+\frac{n}{2})\right]^{-2}=(2\pi)^{-n}\omega_n^2. $

因此,$\eta(t)$$(0,(2\pi)^{-n}V(\Omega)]$上是递减的.

至此,定理的证明已完成.

参考文献

Weyl H.

Das asymptotische verteilungsgesetz der eigenwerte linearer partieller differentialgleichungen

Math Ann, 1912, 71: 441-479

DOI:10.1007/BF01456804      URL     [本文引用: 1]

Pólya G.

On the eigenvalues of vibrating membranes

Proc Lond Math Soc, 1961, 11: 419-433

[本文引用: 3]

Li P, Yau S T.

On the Schrödinger equation and the eigenvalue problem

Comm Math Phys, 1983, 88: 309-318

DOI:10.1007/BF01213210      URL     [本文引用: 3]

Berezin F A.

Covariant and contravariant symbols of operators

Izv Akad Nauk SSSR Ser Mat, 1972, 36: 1134-1167

[本文引用: 6]

Lieb E.

The number of bound states of one-body Schroedinger operators and the Weyl problem

Proc Sym Pure Math, 1980, 36: 241-252

[本文引用: 2]

Laptev A.

Dirichlet and Neumann eigenvalue problems on domains in Euclidean spaces

J Funct Anal, 1997, 151: 531-545

DOI:10.1006/jfan.1997.3155      URL     [本文引用: 2]

Melas A D.

A lower bound for sums of eigenvalues of the Laplacian

Proc Amer Math Soc, 2002, 131: 631-636

DOI:10.1090/S0002-9939-02-06834-X      URL     [本文引用: 15]

Cheng Q M, Yang H C.

Estimates for eigenvalues on Riemannian manifolds

J Differ Equations, 2009, 247: 2270-2281

DOI:10.1016/j.jde.2009.07.015      URL     [本文引用: 1]

Yolcu S Y, Yolcu T.

Multidimensional lower bounds for the eigenvalues of Stokes and Dirichlet Laplacian operators

J Math Phys, 2012, 53: 043508

DOI:10.1063/1.3701978      URL     [本文引用: 8]

Yolcu S Y, Yolcu T.

Refined eigenvalue bounds on the Dirichlet fractional Laplacian

J Math Phys, 2015, 56: 1-12

[本文引用: 8]

Davies E B, Safalov Y. Spectral Theory and Geometry. Cambridge: Cambridge University Press, 1999

[本文引用: 1]

Wei G X, Sun H J, Zeng L Z.

Lower bounds for fractional Laplacian eigenvalues

Commun Contemp Math, 2014, 16: 1450032

DOI:10.1142/S0219199714500321      URL     [本文引用: 1]

/